GENERAL PRACTITIONER EXAM
Question Summary
0 of 100 questions completed
Questions:
- 1
- 2
- 3
- 4
- 5
- 6
- 7
- 8
- 9
- 10
- 11
- 12
- 13
- 14
- 15
- 16
- 17
- 18
- 19
- 20
- 21
- 22
- 23
- 24
- 25
- 26
- 27
- 28
- 29
- 30
- 31
- 32
- 33
- 34
- 35
- 36
- 37
- 38
- 39
- 40
- 41
- 42
- 43
- 44
- 45
- 46
- 47
- 48
- 49
- 50
- 51
- 52
- 53
- 54
- 55
- 56
- 57
- 58
- 59
- 60
- 61
- 62
- 63
- 64
- 65
- 66
- 67
- 68
- 69
- 70
- 71
- 72
- 73
- 74
- 75
- 76
- 77
- 78
- 79
- 80
- 81
- 82
- 83
- 84
- 85
- 86
- 87
- 88
- 89
- 90
- 91
- 92
- 93
- 94
- 95
- 96
- 97
- 98
- 99
- 100
Information
Hi, Welcome to General Practioner Exam
You have already completed the Exam before. Hence you can not start it again.
Exam is loading...
You must sign in or sign up to start the Exam.
You have to finish following Exam, to start this Exam:
Results
0 of 100 questions answered correctly
Your time:
Time has elapsed
You have reached 0 of 0 points, (0)
Average score |
|
Your score |
|
Categories
- Not categorized 0%
Pos. | Name | Entered on | Points | Result |
---|---|---|---|---|
Table is loading | ||||
No data available | ||||
- 1
- 2
- 3
- 4
- 5
- 6
- 7
- 8
- 9
- 10
- 11
- 12
- 13
- 14
- 15
- 16
- 17
- 18
- 19
- 20
- 21
- 22
- 23
- 24
- 25
- 26
- 27
- 28
- 29
- 30
- 31
- 32
- 33
- 34
- 35
- 36
- 37
- 38
- 39
- 40
- 41
- 42
- 43
- 44
- 45
- 46
- 47
- 48
- 49
- 50
- 51
- 52
- 53
- 54
- 55
- 56
- 57
- 58
- 59
- 60
- 61
- 62
- 63
- 64
- 65
- 66
- 67
- 68
- 69
- 70
- 71
- 72
- 73
- 74
- 75
- 76
- 77
- 78
- 79
- 80
- 81
- 82
- 83
- 84
- 85
- 86
- 87
- 88
- 89
- 90
- 91
- 92
- 93
- 94
- 95
- 96
- 97
- 98
- 99
- 100
- Unanswered
- Answered
- Review
-
Question 1 of 100
1. Question
1 pointsA 73-year-old male presented with an acute attacks of gout in his left knee. What is the most likely underlying metabolic cause?
Correct
Incorrect
Explanation:
The aetiology of gout can broadly be divided into cases where there is underexcretion of urate via the kidney (90%) or endogenous overproduction of uric acid (10%) although in practical terms the distinction is rarely made as it allopurinol is the mainstay of long- term treatment (not during the acute attack!) in both groups. In a 73- year-old man it is almost certainly reduced renal excretion due to deteriorating renal function and possibly diuretic use. Excessive dietary intake of purines is unlikely to be the main cause in this case.
-
Question 2 of 100
2. Question
1 pointsA 63 year old man undergoes screening colonoscopy, which demonstrates a 7 mm-diameter polyp in the rectum. Biopsy shows glands and sawtooth crypts composed of a proliferation of goblet and columnar epithelial cells. No cellular atypia is seen. What would be the best classification of this polyp?
Correct
Incorrect
Explanation:
This is a hyperplastic polyp; these polyps comprise 90% of all colonic polyps and have no malignant potential.
Peutz-Jeghers polyps (choice B) also have no malignant potential, but tend to be larger and hive a complex branching pattern.
Tubular adenomas, tubulovillous adenomas, and villous adenomas (choices C, D, and E) are all true neoplastic polyps containing dysplastic epithelium; the malignant potential of these polyps increases with size and the percentage of the polyp that has a villous configuration. -
Question 3 of 100
3. Question
1 pointsACTH excess is found in which one of the following condition?
Correct
Incorrect
Explanation:
ACTH excess is often seen in Cushing´s syndrome, but not in Hashimoto´s, Graves´ disease, Marfan´s, or Paget´s.
-
Question 4 of 100
4. Question
1 pointsWhich two organs are primarily affected by Goodpasture´s syndrome?
Correct
Incorrect
Explanation:
Goodpasture´s syndrome affects the lungs and the kidneys. It causes both pulmonary hemorrhage and acute glomerulonephritis because the body recognizes a self antigen (in this case, a basement membrane protein) which elicits an immune response. The other organs are not typically involved.
-
Question 5 of 100
5. Question
1 pointsA diabetic male aged 56 years presents with symptoms of intermittent claudication of the buttocks and thighs. He also complained of impotence. On examination this patient had signs of peripheral vascular disease. His femoral pulses were not palpable. The most likely diagnosis is
Correct
Incorrect
Explanation:
Leriche syndrome is caused by atherosclerosis resulting in obstruction of the aortic bifurcation, and causes intermittent claudication and impotence. Atheroclerosis is more common in diabetics and smokers. Takayasu´s disease is common in young women. It involves arteritis of the aorta and its major branches. It is also called “pulseless disease.” These patients present with weight loss, anorexia, loss of pulses in the upper extremities, and, sometimes, cerebral ischemia. Cystic medial necrosis is associated with ascending aortic dissection. Thromboangiitis obliterans occurs in young men who are heavy smokers. An inflammatory reaction develops in veins and small arteries leading to superficial thrombophlebitis and arterial obstruction with ulceration or gangrene of the digits. Raynaud´s phenomenon is a vasospastic disorder in which cold exposure causes blanching of the fingers followed by cyanosis in some cases. It can occur secondary to scleroderma.
-
Question 6 of 100
6. Question
1 pointsAn alcoholic is taken to the emergency department in respiratory distress. CXR shows lobar consolidation of the right lower lung. The infectious agent that should be highest on the differential diagnosis is which one of the following?
Correct
Incorrect
Explanation:
Lobar pneumonia, in which an entire lobe of the lung becomes rapidly affected with pneumonia, is actually a relatively uncommon pattern for pneumonia. Common causative organisms include Streptococcus pneumoniae (pneumococcus), Haemophilus influenzae, and Klebsiella pneumoniae. K. pneumoniae is specifically associated with alcohol abuse, diabetes mellitus, and nosocomial infections. Legionella spp. most commonly causes a patchy atypical pneumonia not specifically associated with alcoholism. Instead, Legionella infection is associated with inspiration of aerosolized contaminated water. Mycoplasma pneumoniae causes atypical pneumonia (extensive patchy infiltrates) rather than lobar pneumonia. It is the most common cause of pneumonia in young adults. Pneumocystis carinii causes atypical pneumonia with diffuse interstitial and alveolar infiltrates, typically in patient´s immunosuppressed by AIDS, cytotoxic drug therapy, or cancer. Staphylococcus aureus produces a necrotizing, abscessing pneumonia. Staphylococcal pneumonia may complicate influenza during epidemics, or it may be a nosocomial infection. It is associated with empyema formation. -
Question 7 of 100
7. Question
1 pointsA patient is brought into the emergency department with lethargy and confusion. He is type 2 diabetic and is currently taking metformin. He also admits to frequent consumption of alcohol. Laboratory values show:
- PaO2 = 90mmHg
- PaCO2=19mmHg
- pH=7.30
Assuming a respiratory exchange ratio of 1, which pulmonary condition is most likely to be present?
Correct
Incorrect
Explanation:
The alveolar PO2 is found from the alveolar gas equation:
PAO2 = PIO2 – PACO2 /R
Plo2 = .21 (P barometric – P water vapor)
P barometric = 760 mm Hg
water vapor = 47
then P102 = (.21)(713) = 149mm Hg. Therefore PAQ2 = 149-(19/1) = 130mm Hg. Thus, the alveolar-arterial difference is 130-90 =40 suggesting some underlying lung disease that is impairing oxygen uptake in the lungs.
Option A is incorrect because the decreased arterial PCO2 indicates increased alveolar ventilation. Option B is incorrect because only severe hypoxia (PO2 < 50 mm Hg) would cause alveolar ventilation to double, as it has in this patient. Primary respiratory acidosis would display increased, rather than decreased, arterial PO2. Acidosis shifts the O2 dissociation curve to the right, increasing the P50. -
Question 8 of 100
8. Question
1 pointsA 4 year old girl is brought because her mother has noticed that she has started to “swell up” and her urine leaves lots of “bubbles” in the toilet. Exam reveals generalized edema, especially noticeable in dependent regions and under the eyes. Which urinary constituent is probably presents in abnormally high concentration and accounts for the bubbles?
Correct
Incorrect
Explanation:
The combination of edema and frothy urine suggests nephrotic syndrome. Nephrotic syndrome is a glomerular disease characterized by severe proteinuria (>3.5 g/day), hypoalbuminemia (<3 g/dL), generalized edema, hyperlipidemia, and lipiduria. Urine with high protein content is more able to form stable bubbles than in normal urine, and maybe a clue that a patient (or parent) notices. Chloride in urine cannot be detected grossly, and would not cause frothy urine. Some people can detect a sweet smell to urine that contains large amounts of glucose and Greek physicians were known to taste urine to diagnose diabetes, but the presentation here suggests nephrotic syndrome. Sodium in urine cannot be detected grossly and would not cause frothy urine. Very concentrated urine with high urea content looks darker in color, and is not necessarily frothy.
-
Question 9 of 100
9. Question
1 pointsA 54 year old man has vomiting. He has a long history of alcohol abuse, appears slightly jaundiced and is disheveled and unkempt. IV glucose infusion and diazepam were started and he symptomatically improved. A day later he becomes confused, develops vomiting, diplopia and is unable to stand. The most likely diagnosis is which one of the following?
Correct
Incorrect
Explanation:
This patient is manifesting signs of Wernicke´s encephalopathy with confusion, oculomotor signs and ataxia affecting gait and stance. Wernicke´s encephalopathy is a medical emergency, requiring urgent intravenous thiamine. The episode has been precipitated by intravenous dextrose administration which has exhausted his vitamin B reserves; hence B vitamins must be administered to all alcoholic patients requiring dextrone.
-
Question 10 of 100
10. Question
1 pointsA boy aged 7 years is admitted with renal colic due to renal calculus. His mother has a similar history of recurrent calculi. The most likely explanation for recurrent renal calculi in both mother and child is which one of the following?
Correct
Incorrect
Explanation:
Idiopathic hypercalciuria has a familial or sporadic pattern. In the familial pattern an autosomal dominant inheritance is present. The type of the disease is identical in affected members of the same family and the typical presentation is of recurrent urinary calculi. Cystinuria, cystinosis, urate uropathy and hyperoxaluria are autosomal recessive conditions.
-
Question 11 of 100
11. Question
1 pointsIn Marfan syndrome, mutations in which of the following is present?
Correct
Incorrect
Explanation:
Marfan syndrome occurs due to a mutation in the fibrillin gene. Most patients, who are prone to develop an aortic aneurysm as a component of Marfan syndrome, can be identified by detection of mutations in the fibrillin-1 gene.
Patients with the rarer form of Marfan syndrome, which is characterised by contracturalarachnodactyly instead of loose joints, can usually be identified by detection of a mutation in the Eibrillin-2 gene that is similar in structure to the gene for fibrillin-l.
Preliminary data suggest that patients with mutations in the Fibrillin-2 gene are not prone to develop aneurysms. -
Question 12 of 100
12. Question
1 pointsGaucher´s disease is associated with the deficiency of:
Correct
Incorrect
Explanation:
Hexosaminidase A deficiency is associated with Tay-Sachs disease. Sphingomyelinase deficiency is associated with Niemann Pick disease. Arylsulphatase-A deficiency is associated with metachromic leucodystrophy.Iduronidase deficiency is associated with Hurlers syndrome.
-
Question 13 of 100
13. Question
1 pointsA 34-year-old smoker is diagnosed with a vasculitis. Serum p-ANCA is raised along fibrinoid necrosis of small arteries. Which one of the following represents the most likely diagnosis?
Correct
Incorrect
Explanation:
Classic polyarteritis nodosa is a vasculitis associated with involvement of small arteries and p-ANCA antibodies. While smoking is associated with thromboangiitis obliterans (Buerger´s Disease), it is not associated with an increase in autoantibodies. Wegener´s granulomatosis is known to be associated with c-ANCA autoantibodies. The presence of c-ANCA antibodies is highly suggestive of a diagnosis of Wegener´s granulomatosis. Temporal and Takayasus arteritis are not associated with autoantibodies.
-
Question 14 of 100
14. Question
1 pointsAn inherited disorder that can produce sudden death is
Correct
Incorrect
Explanation:
Gilbert´s syndrome (C) is decreased uridine diphosphaste (UDP) gluconryl transferase activity, which produces a mild UCBQ Ward-Romano syndrome (E) is a prolonged QT syndrome leading to severe and fatal arrythmias. Crigler-Najjar syndrome, type I (A), is an absence of UDP glucuronyl transferase activity and is usually fatal in infancy. Dubin-Johnson (D) and Rotor´s (B) syndromes are defects in transport of conjugate bilirubin out of the hepatocyte. Dubin-Johnson´s differs from Rotor´s in that the hepatocytes are pigmented in Dubin-Johnson´s and not in Rotor´s.
-
Question 15 of 100
15. Question
1 pointsWhich of the following is the causative agent of Lyme disease?
Correct
Incorrect
Explanation:
Only Borrelia burgdorferi is known to cause Lyme disease.
-
Question 16 of 100
16. Question
1 pointsWhich of the following statement does NOT characterize secretory diarrhea?
Correct
Incorrect
Explanation:
Bile acid induced diarrhea following cholecystectomy may occur, due to the loss of storage capacity of the full bladder. Other causes of bile acid malabsorption of the distal ileum may also produce the diarrhea. Secretory diarrhea is characterized by a large volume of fecal output caused by abnormal fluid and electrolyte transport. It is usually not related to the ingestion of food and therefore persists despite fasting. Because there is no malabsorbed solute, fecal osmolality can be accounted for by normal ionic constituents. The classic examples of secretory diarrhea, such as that occurring with carcinoid, Zollinger Ellison syndrome, pancreatic cholera, and medullary carcinoma are those mediated by hormones.
-
Question 17 of 100
17. Question
1 pointsInfection with Calymmatobacterium granulomatis causes which one of the following?
Correct
Incorrect
Explanation:
Infection with Calymmatobacterium granulomatis causes painful beefy red granulomatous lesions. It is a sexually transmitted disease. H. ducreyi causes chancroid, which presents with a painful ulcer and secondary inguinal adenopathy. Treponema pallidum causes syphilis. Gardenerella spp. is a factor in bacterial vaginosis. Chlamydia trachomatis subtypes L1-L3 may cause lymphogranuloma venereum, which may produce a vesicular lesion followed by inguinal lymphadenitis and, eventually bubo inguinale.
-
Question 18 of 100
18. Question
1 pointsA 30-year-old male presents to you seeking advice regarding starting a family. He has common variable immunodeficiency and wants to know that what is the risk of passing this on to his children?
Correct
Incorrect
Explanation:
Common variable immunodeficiency involves low levels of most or all of the immunoglobulin classes, a lack of B lymphocytes or plasma cells that are capable of producing antibodies, and is associated with frequent bacterial infections. The cause of CVID is unknown but a family member may be affected in approximately 20%, but there is no clear pattern of inheritance.
-
Question 19 of 100
19. Question
1 pointsWhich of the following may be responsible for a hypokalaemic hypertension?
Correct
Incorrect
Explanation:
Liddle´s syndrome is typically associated with hypokalaemic hypertension and low renin and aldosterone concentrations – the so- called pseudo-hyperaldosteronism. Bartter´s syndrome is associated with hypokalaemia, though hypertension is not a feature.
In type IV RTA, there is a hyporeninaemichypoaldosteronism, which may also be produced with diabetic nephropathy, hence hyperkalaemia is more typical. -
Question 20 of 100
20. Question
1 pointsAll of the following are strong determinants of development of type I diabetes mellitus EXCEPT
Correct
Incorrect
Explanation:
Type I diabetes mellitus results from destruction of pancreatic beta cells. Identical twins show concordance rates of 30-50%. Because concordance is not 100%, dietary factors, such as milk protein and toxins, have also been implicated. The most attention has been given to viral infections, such as coxsackie and rubella, which have been associated with increased frequency of type I diabetes. Mononuclear inflammation of pancreatic islets alters beta cells so that they are identified as foreign. Cytotoxic cells cause cell death of beta, but neither alpha nor delta cells are damaged. Belly fat is related strongly to type II and not to type I diabetes.
-
Question 21 of 100
21. Question
1 pointsWhich of the following is NOT correct association between molecular genetic abnormalities and colon adenomas/carcinomas?
Correct
Incorrect
Explanation:
As currently understood, a series of genetic changes occur in concert to initially produce an adenoma that may subsequently progress to a carcinoma. The initial step probably involves a mutation of the K-ras proto-oncogene. The gene for familial adenomatous polyposis has been mapped to chromosome 5. Chromosome 17(p53 gene) and chromosome 18 (DDC gene) deletions are thought to be important in malignant transformation. Overexpression of the c-myc gene has been reported in colon cancers.
-
Question 22 of 100
22. Question
1 pointsThe thalassemias are caused by which one of the following?
Correct
Incorrect
Explanation:
he thalassemia family is characterized by a congenital disorder in synthesis of one or more of the subunits of hemoglobin. Increased hemolysis and splenic sequestration may be found as a result of abnormal red cells, but they are not causes of thalassemia. Thalassemia is not caused by blood loss or sulfonamides.
-
Question 23 of 100
23. Question
1 pointsWhich of the following has NOT been implicated as an increased risk factor for the development of acalculous cholecystitis?
Correct
Incorrect
Explanation:
In approximately 10% of cases of acute cholecystitis, no gallstones are present at surgery. An increased risk for acalculous cholecystitis is associated with severe burns and significant trauma. Ischemia resulting from also produces acalculous cholecystitis. Direct infection of the gallbladder with “unusual” organisms, such as Leptospira, Salmonella, or Vibrio cholerae, is also associated with acalculous cholecystitis. Acute calculus cholecystitis is most often associated with infections caused by Escherichia coli and other enteric Gram negative organisms. Systemic diseases, such as sarcoidosis, syphilis, and tuberculosis have also been associated with acalculous cholecystitis.
-
Question 24 of 100
24. Question
1 pointsTrue statement concerning infection with hepatitis D virus is
Correct
Incorrect
Explanation:
Hepatitis D, also known as the delta virus, is a defective RNA virus, which requires the assistance of either hepatitis B or other hepadnaviruses for its replication and expression. In areas of high prevalence, transmission is person to person. In areas of low prevalence, transmission is typically among groups with exposure either to blood products or contaminated needles. Due to its reliance on hepatitis B, the duration of infection of hepatitis D parallels that of hepatitis B. Genomic heterogeneity among hepatitis D isolates has no clinical or pathologic implications. Concomitant infection with hepatitis B and D does not increase the probability of chronic hepatitis, but does increase the likelihood of fulminant hepatic failure as compared to those persons with hepatitis B infection alone.
-
Question 25 of 100
25. Question
1 pointsA female aged 26 years with sclerodactyly, telangiectases, and Raynaud´s phenomenon presents with cramping diarrhea. Which one of the following is the most likely diagnosis in this patient?
Correct
Incorrect
Explanation:
The patient is likely to have scleroderma (progressive systemic sclerosis), which is often associated with a blind loop syndrome due to abnormal motility of the intestine. This constellation of symptoms is not characteristic of diverticulitis, appendicitis, gastroesophageal reflux, or Crohn´s disease.
-
Question 26 of 100
26. Question
1 pointsWhich one of the following is the most common underlying condition in patients with an episode of acute pancreatitis?
Correct
Incorrect
Explanation:
Abdominal trauma and hyperparathyroidism are uncommon causes of acute pancreatitis; diabetes mellitus and ulcers may develop in patients with pancreatitis, rather than the reverse.
-
Question 27 of 100
27. Question
1 pointsA female patient is found to be 46, XY. Her vagina is very shallow, ending in a blind pouch, and there are palpable masses in the labia. Testicular feminization syndrome is diagnosed. Most likely factor present during the early fetal life is
Correct
Incorrect
Explanation:
Testicular feminization is a disorder of the androgen receptor. Phenotypically, the patient appears female, but has a blindly ending vagina and lacks a uterus or other female internal reproductive organs. The patient has an XY genotype. Since the gene for testes determining factor (TDF) is on the Y chromosome, TDF will cause the indifferent gonad to develop into a testis containing Sertoli cells. Sertoli cells at this stage will secrete Mullerian inhibitory factor (MIF), a substance that suppresses the paramesonephric ducts, preventing the formation of female internal reproductive organs. This patient would not have a streak ovary, a finding in Turner syndrome that is associated with a 45, XO genotype. In fact, the patient would have testes, since the genetic complement contained a Y chromosome. The testes in individuals with testicular feminization syndrome are often undescended and are usually removed surgically. This patient would not possess a uterus or an oviduct. Both of these structures are derived from the paramesonephric duct, which is suppressed by MIF. Depressed levels of testosterone would not occur in this patient. In fact, individuals with testicular feminization syndrome have normal or even slightly elevated levels of testosterone. The development of female external genitalia is the result of defective androgen receptors, not depressed levels of testosterone.
-
Question 28 of 100
28. Question
1 pointsA nurse receives an intradermal inoculation of tuberculin during a pre-employment examination. During her employment in Pakistan, she was vaccinated with BCG. There is an 8 mm zone of Erythema and induration over the injection site at 48 hours. The most important costimulatory signal involved in the elicitation of this response is
Correct
Incorrect
Explanation:
The tuberculin response is the classic example of type IV hypersensitivity, which is mediated by TH1 cells and macrophages. The initial recognition signal involves interaction between MHC class II and the T cell receptor. The B7 molecule on the cell surface of the antigen presenting cell reacts with the CD 28 molecule on the T cell surface for maximal costimulatory signals. The B7 molecule on the surface of the antigen presenting cell reacts only with CD 28 and does not react with LFA-1 adhesion molecule. The ICAM-I on the surface of an antigen presenting cell reacts with the LFA- I on the surface of a T cell for the purpose of cell-to-cell adhesion and does not function for costimulation. LFA-3 (CD58) is an adhesion molecule on the surface of an antigen presenting cell. It does not react with a CD28 costimulatory molecule on the T cell surface. The MHC class II molecule with its epitope does interact with a specific T cell receptor (TCR) but this is not termed costimulatory. However, the interaction does stimulate the T cell to produce interleukins for further cell division.
-
Question 29 of 100
29. Question
1 pointsFour of the six family members of a Malaysian family have displayed increased susceptibility to extracellular bacteria and elevated serum levels of IgM and IgG. Serum complement protein analysis reveals near absence of component C4. Which complement activity would be inhibited by this deficiency?
Correct
Incorrect
Explanation:
The classic complement pathway involves C1, C4, and C2 to the point of splitting C3. C3 is then split to yield C3b and C3a. A deficiency of C4 would have no effect on any of the other answer choices listed. The complement protein C3 can be split into C3a and C3b using the alternative pathway. The additional proteins required in this pathway would be factors B and D and properdin. The C5 convertase enzyme can be formed in the alternative pathway without using C4. The C5 molecule could be split into C5a and C5b using the alternative pathway, without the involvement of C4. The membrane attack complex (C5b, C6, C7, C8, and polymers of C9) lyses the pathogenic cell. This process does not require the alternative pathway and would not require C4.
-
Question 30 of 100
30. Question
1 pointsA 36 year old G5P0 woman is referred to a specialist to examine the cause of her multiple miscarriages. Most miscarriages occurred during the first trimester. She has no significant past medical history, and no contributing family history of infertility. Which statement best accounts for her fertility problems?
Correct
Incorrect
Explanation:
Recurrent miscarriages in the first trimester are usually indicative of a chromosomal abnormality. Among these Robertsonian translocations are most common. Major chromosomal deletions or genetic mutations are unlikely since the carrier (the mother) appears to be healthy. Robertsonian translocations involving chromosome 14 and 21, for example, are not only a common cause of Down syndrome, but may also lead to recurrent abortions. While advanced maternal age (greater than 35 years old) may increase the risk of Down syndrome, several consecutive first-trimester abortions are unlikely. Paternal age is rarely a cause of recurrent abortions. She cannot have an autosomal monosomy because that would be incompatible with life. A person with a bicornuate uterus may also suffer from recurrent abortions, although recurrent first trimester abortions are most likely related to a chromosomal abnormality.
-
Question 31 of 100
31. Question
1 pointsA patient undergoes esophagogastroduodenoscopy. He has prolonged diarrhea. Biopsy of the small intestine shows numerous crescent shaped protozoa adjacent to the epithelial brush border. Which organism is most likely involved?
Correct
Incorrect
Explanation:
The probable organism is Giardia lamblia which characteristically infects the small intestine. When seen in smears from duodenal aspirates, this flagellated organism has a characteristic “face like” appearance. However, in biopsy specimens the organism is often caught at an angle, and the characteristic appearance and location is as described in the question stem. Giardia is a common contaminant of water supplies, and patients who ingest the cysts may be asymptomatic or may occasionally develop prolonged diarrhea or intestinal malabsorption. Some patients with severe disease have low serum IgA or low overall immunoglobulin levels. Entamoeba histolytica usually infects the large intestine and/or liver. Escherichia coli is a bacterial cause of diarrhea. Naegleria fowleri causes meningoencephalitis. Trichomonas vaginalis causes vaginitis.
-
Question 32 of 100
32. Question
1 pointsA 25 year old man has of multiple small nodules on his lips. The physician notes that he has a tall and thin build with long fingers. In addition, he has hyperextensible joint. Which screening test is indicated?
Correct
Incorrect
Explanation:
The presence of mucosal neuromas, particularly when multiple and when the patient has a marfanoid syndrome, is a marker for multiple endocrine neoplasia type IIb (MEN IIb). MEN IIb predisposes for medullary carcinoma of the thyroid and pheochromocytoma. Pentagastrin stimulated calcitonin studies, may suggest the presence of thyroid C cell hyperplasia, which may precede frank medullar carcinoma. High serum gastrin suggests pancreatic or duodenal gastrinomas, which may be a component of MEN I. High serum insulin or vasoactive intestinal peptide also suggest pancreatic endocrine tumors, which maybe a component of MEN I. Parathyroid hyperplasia or adenomas with parathyroid hormone secretion can be seen in MEN I and MEN IIa, but are not part of MEN IIb.
-
Question 33 of 100
33. Question
1 pointsA 28 year old male has heartburn for 1 week. Antacids did not provide relief. Endoscopy reveals punched out ulcers in the lower segment of the esophagus. Biopsy reveals chronic inflammation and epithelial cells with glassy intranuclear inclusions. Which is the most likely cause of this condition?
Correct
Incorrect
Explanation:
Herpes simplex is one of the most common etiologic agents of infective esophagitis. The disease may manifest in immunocompetent hosts, although it is more frequent in immunocompromised (e.g., AIDS) patients. Punched out ulcers are highly characteristic. The histology is rather nonspecific (chronic inflammatory infiltrate) except for the presence of glassy intranuclear eosinophilic inclusions Cytomegalovirus is the other common viral etiology of esophagitis.-This form would be characterized by purple intranuclear inclusions combined with granular cytoplasmic inclusions, and markedly enlarged cell size.
Candida albicans gives rise to esophagitis in immunocompromised hosts or develops superimposed on CMV/herpes ulcers. Candida organisms are easily identified admixed with squamous cells and neutrophils. Chemical injury is usually due to accidental or suicidal ingestion of corrosive agents, such as lye acids, and detergents. The mucosa shows diffuse erythema, edema, and (in severe cases) extensive ulcerations. The intestinal microbe Escherichia coli does not usually cause disease in the esophagus. Helicobacter pylori does not cause esophagitis. This gram negative organism is associated with chronic gastritis of the antrum, peptic ulcer, gastric adenocarcinoma, and lymphoma. -
Question 34 of 100
34. Question
1 pointsWhich of the following statements concerning the thymus is true?
Correct
Incorrect
Explanation:
Cortical thymocytes are immature forms, and either do not express CD4 or CD8 (double negative cells) or express both CD4 and CD8 (double positive cells). As the cells mature, they pass to the thymic medulla where they lose expression of either CD4 or CD8, to become single positive cells.
Negative selection occurs at the stage when thymocytes express both CD4 and CD8, but co-expression of these markers does not mediate negative selection. Negative selection occurs when a thymocyte expresses a TcR with high affinity for self-antigen: MHC complexes in the thymic micro-environment.
Once a thymocyte has successfully rearranged and expressed an -alpha/beta or gamma/delta TcR it is committed to that lineage.
Thymocytes whose TcR bind with high affinity to self Ag/MHC complexes are clonally deleted by a process of negative selection. B cells express IgM and IgD; T cells do not! -
Question 35 of 100
35. Question
1 pointsAn old man who lives alone and prepares his own food is found to have numerous ecchymotic hemorrhage areas around his fragmented hair follicles. Hematomas are also found in limb muscles. Except for the absence of teeth, the examination is normal. PT, APTT and full blood count is normal except for a hematocrit of 28%. What deficiency is present?
Correct
Incorrect
Explanation:
This man has features of scurvy. Scurvy is the clinical state arising from dietary deficiency of vitamin C (ascorbic acid). It results in impaired collagen synthesis. The typical pathological manifestations of vitamin C deficiency are noted in dentine, osteoid and capillary vessel wall tissues (Clinical features include gum swelling, friability, bleeding, and infection with loose teeth; mucosal petechiae; scleral icterus (late, probably secondary to hemolysis); and pale conjunctiva are seen. Fractures, dislocations, and tenderness of bones are common in children. Bleeding into muscles and joints may be seen. Perifollicular hyperkeratotic papules, perifollicular hemorrhages, purpura, and ecchymoses are the classical skin manifestation of scurvy.
-
Question 36 of 100
36. Question
1 pointsA 26-year-old man presents with proteinuria, haematuria and sensorineural deafness. Which of the following protein structures is likely to be abnormal?
Correct
Incorrect
Explanation:
The diagnosis is Alport´s syndrome, which is a disorder of Type 4 collagen assembly and is inherited as an X-linked (most likely to recessive) disorder in 85% of cases. Fibrillin gene abnormalities are associated with Marfan´s syndrome. Type 1 collagen disorders are associated with osteogenesisimperfecta; it is the main type of collagen in tendon and bone. Type III Collagen is the main component of reticular fibres.
-
Question 37 of 100
37. Question
1 pointsWhich one of the following is a feature of the VIPoma syndrome?
Correct
Incorrect
Explanation:
All others are opposite to what is expected.
The features of VIPoma include)- Vasoactive intestinal polypeptide secreting tumour
- Mainly pancreas rarely ganglioneuroblastoma (sympathetic chain adrenal cortex)
- Diarrhoea (´pancreatic cholera´)
- Weight loss
- Dehydration
- Abdominal colic
- Cutaneous flushing
- raised plasma vasoactive intestinal polypeptide (VIP)
- Urea+calcium
- Raised plasma pancreatic polypeptide
- Hypokalaemic acidosis (loss of alkaline secretions)
- Achlorydia
- Mildly raised glucose
- normal functions of VIP
- Increased intestinal secretion of water and electrolytes
- Peripheral vasodilation
- Inhibits gastric acid secretion
- Potentiates acetylcholine action on salivary glands.
-
Question 38 of 100
38. Question
1 pointsA 35-year-old male presents with severe abdominal pain, nausea, vomiting, and dehydration. On exam, his bowel sounds are diminished and he feels slightly relieved on his left lateral fetal position. He is not jaundiced. On lab data, the lipase level is elevated. Which of the following is NOT associated with his condition?
Correct
Incorrect
Explanation:
This patient has an acute episode of pancreatitis, which can be caused by alcohol, gallstones, blunt abdominal trauma, and medications, such as Lasix, DDI, and chemotherapy and it also has a heritable factor. Pancreatic cancer usually presents as asymptomatic jaundice, weight loss, and depression. The lipase level is not usually elevated in pancreatic cancer.
-
Question 39 of 100
39. Question
1 pointsA young man during heavy intoxication severely injures his right wrist by breaking through a glass window. In the emergency room, the physician assesses the integrity of the ulnar nerve at the wrist by asking the patient to perform which one of the following maneuvers?
Correct
Incorrect
Explanation:
Ulnar nerve injury at the wrist (median cord C8-T1) from a slashing of the wrist results in loss of ability for adduction of the thumb due to the paralysis of the adductor pollicis muscle.
-
Question 40 of 100
40. Question
1 pointsWhich one of the following is the most common autoimmune disorder?
Correct
Incorrect
Explanation:
Rheumatoid arthritis affects one percent of the American population, rendering it the most common autoimmune disease.
Autoimmune Disease in USA
Rheumatoid Arthritis: > 2 million
Diabetes Mellitus Type I: >1.5 million
Inflammatory Bowel Disease: >1.4 million -
Question 41 of 100
41. Question
1 pointsThe most commonly mutated gene in the progression of human cancer is which one of the following?
Correct
Incorrect
Explanation:
p53, a tumor suppressor gene, is found to be mutated in over 50% of all types of human cancers. -
Question 42 of 100
42. Question
1 pointsA 33 year old HIV positive male presents with sudden onset of right sided hemiplegia. Brain CT scan with contrast reveals a ring enhancing lesion in the left parietal lobe. CMV antibody is positive. The most likely diagnosis is Correct
Incorrect
Explanation:
CNS toxoplasmosis infection is the most common cause of a ring-enhancing lesion in the brain of an AIDS patient. Five percent of patients will not have a positive toxoplasmosis anti- body titer in the serum. A focal neurologic deficit is the most common initial presentation. Cryptococcal meningitis presents with nuchal rigidity, fever, and photophobia. It would not present as a ring enhancing lesion. Tuberculoma would be rare unless other signs of tuberculosis are present. Cerebral infarct would not be common in someone this young. Hodgkin´s lymphoma usually spreads to the hilar and retro peritoneal lymph nodes. CNS non Hodgkin´s lymphoma occurs in 25% of AIDS patients with a ri
-
Question 43 of 100
43. Question
1 pointsA 33 year old man presents with right sided rib pain. Past medical history is not significant. CXR was unremarkable, but a follow up bone scan revealed a single osteolytic lesion on the right fifth rib. Biopsy showed foamy macrophages admixed with eosinophils. The most likely diagnosis is
Correct
Incorrect
Explanation:
This patient has unifocal Langerhans cell or eosinophilic granulomatosis. This benign disorder is characterized by a single osteolytic lesion either in the rib or skull. Biopsy would show Langerhans histiocytes along with eosinophils. Many lesions are asymptomatic. Multiple myeloma patients would also have lytic lesions in the ribs, but plasma cells are found on biopsy of the bone marrow, not histiocytes and eosinophils. Hodgkin´s disease is characterized by lymphadenopathy and Reed Sternberg cells on the bone marrow biopsy. Hand Schuller Christian disease is a subset of multifocal eosinophilic granulomatosis. It is associated with a triad of lytic skull lesions, exophthalmos, and diabetes insipidus. Letterer Siwe syndrome is a disorder of infants. It is associated with lymphadenopathy and hepatosplenomegaly. -
Question 44 of 100
44. Question
1 pointsA 55 year old man presents with fatigue and weight loss of 30 pounds in the past month. On examination marked splenomegaly is revealed. Bone marrow yields a “dry tap.” Lymphoid cells obtained during this aspiration are positive for tartrate resistant acid phosphatase and CD25 surface antigen. The patient is most likely suffering from?
Correct
Incorrect
Explanation:
Hairy cell leukemia is a B-cell lymphoma, which is characterized by tumorigenic production of B-cells, which have projections that are analogous to hairs on at scalp. Hairy cell leukemia cells are TRAP (tartate-resistant acid phosphatase) positive and CD25 positive. Typically, bone marrow aspiration yields a “dry tap” owing to increased marrow reticulin content. Patients are often anemic in hairy cell leukemia, explaining the fatigue of the patient described in the question. None of the other lymphoid disorders present with the same types of laboratory test results.
-
Question 45 of 100
45. Question
1 pointsMycoplasma pneumoniae accounts for up to 20% of all pneumonias in children and young adults. Which of the following is FALSE regarding mycoplasmas?
Correct
Incorrect
Explanation:
The Mycoplasmas are the smallest free living organisms. They are bound by a triple layer membrane and have no cell wall. The lack of a cell wall prevents visualization on a Gram stain. The lack of a cell wall further prevents effective therapy with antibiotics which are cell wall active, such as beta lactams and vancomycin. Mycoplasma has a wide range of immunomodulatory effects, including stimulation of T and B lymphocytes, stimulating cytokine production, and producing chemotactic factors. These immunomodulatory effects probably account for the appearance of host autoantibodies. Mycoplasma pneumoniae infection is spread by large droplets of respiratory secretions, particularly in closed environments such as households, dormitories, and institutions. The disease is transmitted during the acute phase of infection. Asymptomatic carriers are inefficient transmitters of the disease. Infection spreads slowly through a population because of the prolonged incubation, which usually lasts two to three weeks.
-
Question 46 of 100
46. Question
1 pointsInadequate migration of mesenchymal tissue during fetal growth may lead to
Correct
Incorrect
Explanation:
Extrophy of the bladder usually occurs through protrusion of the posterior urinary bladder into the abdomen; it is caused by migration of mesenchymal tissue to the bladder. Failure of development of kidneys, either unilateral or bilateral, is known as renal agenesis. Double ureter occurs through growth of two ureteric buds or early splitting of one ureteric bud. A patent urachus occurs through the allantois not being destroyed; it may cause diversion of urine through the umbilicus in males with prostatic hypertrophy, A renal cyst occurs as a blind tubule that is fluid filled; it is thought to occur through communication between collecting and distal convoluted tubules in the kidney.
-
Question 47 of 100
47. Question
1 pointsA 32 year old female searches a new boyfriend. Several weeks later, a week after her menstrual period the woman develops a painful, swollen right knee. Organism responsible for this has which of the following characteristics?
Correct
Incorrect
Explanation:
The causative organism is Neisseria gonorrhoeae, which is a sexually transmitted gram negative coccus. It is estimated that N. gonorrhoeae may express over one million variants of pili, resulting in the virtual absence of immunity to re-infection with this agent. The pili of this organism also are protective against phagocytosis and mediate attachment to the mucosal surfaces. The organism is the most common cause of septic arthritis in otherwise healthy, sexually active adults. The treatment is usually with ceftriaxone. The chlamydiae are intracellular organisms that cannot make ATP and do not usually cause septic arthritis. Having both hyphae and spores would be true of Candida, which usually causes arthritis as a rare complication of systemic candidiasis in immunocompromised patients.
Mycoplasma do not have true cell walls and do not usually cause septic arthritis.
Syphilis is caused by a spirochete that is visible only with dark field microscopy or direct fluorescent antibody. Secondary syphilis can (uncommonly) cause an acute arthritis, but the interval between acquiring the infection and developing the arthritis is usually months to years. -
Question 48 of 100
48. Question
1 pointsBiopsy of an ulcerated gastric lesion of a 62 year old smoker shows glands containing cells with enlarged, hyperchromatic nuclei below the muscularis mucosa. Two tripolar mitotic figures are noted. The feature most strongly associated with oncogenesis caused by the most likely organism in this case is
Correct
Incorrect
Explanation:
The patient has gastric carcinoma which has been strongly linked to prior gastric infection with Helicobacter pylori. H. pylori has also been implicated in the etiologies of gastric /peptic ulcer, chronic gastritis, and gastric lymphoma. The disease causing strains of H. pylori contain a pathogenicity island containing the cytotoxin associated gene A (CagA) and a type III secretory system that injects the CagA protein into host cells. ES protein is a product of human papilloma virus that inactivates the p53 tumor suppressor gene and causes oncogenesis in cervical intraepithelial neoplasia. Translocation (8; 14) is the mechanism of oncogenesis in the production of Burkitt lymphoma caused by Epstein-Barr virus. It causes the movement of the c-myc oncogene near the active promoter region for immunoglobulin synthesis. Translocation (11; 18) has been implicated in the formation of gastric lymphomas caused by H. pylori but is not believed to contribute to the formation of gastric carcinomas such as this one.
The VacA gene of H. pylori is associated with pathogenesis, but encodes the vacuolating toxin that causes apoptosis. It is not believed to be involved in the etiology of gastric carcinoma. -
Question 49 of 100
49. Question
1 pointsA 7 year old child develops edema, hypertension and hematuria several weeks after a severe streptococcal sore throat. Best description of the properties of the most likely infectious organism is given by which one of the following? Correct
Incorrect
Explanation:
The most likely cause of glomerulonephritis following sore throat is Streptococcus pyogenes. This is a Group A streptococcus, which is characterized by beta hemolysis and inhibition by the antibiotic bacitracin. Streptococcus pyogenes pharyngitis can also be followed by rheumatic fever. Streptococcus pneumoniae is an example of an alpha hemolytic streptococcus that is inhibited by optochin and lysed by bile. Streptococcus mutans is an example of an alpha hemolytic streptococcus that is not inhibited by Optochin nor lysed by bile. Enterococcus faecalis is an example of a beta hemolytic streptococcus that can grow in 40% bile and 6.5% sodium chloride. Streptococcus agalactiae is an example of a beta hemolytic streptococcus that is resistant to bacitracin and positive for the CAMP test (an incomplete hemolysin). -
Question 50 of 100
50. Question
1 pointsThe type of breast cancer that has the best prognosis is which one of the following?
Correct
Incorrect
Explanation:
Breast adenocarcinoma occurs in a variety of morphologic subtypes some of which have a significantly worse or better prognosis than the most common invasive ductal carcinoma. Tubular carcinoma occurs in women younger than 50 and has an excellent prognosis. It consists of well formed tubules which are so well differentiated that the tumor is sometimes mistaken for a benign lesion. Axillary metastases are present in fewer than 10% of cases at the time of diagnosis. Of the histologic types of breast cancer listed here, the tubular variant is certainly associated with the best chances of survival. Invasive ductal carcinoma and invasive lobular carcinoma are the main morphologic types of breast carcinoma. Together, they constitute 80 to 90% of all cases. The prognosis mainly depends on the staging related to size of the primary cancer and evidence of lymphohematogenous spread to regional nodes and distant organs. Grading (that is, degree of differentiation of primary tumor) has a minor influence on prognosis.
Whenever breast cancer of any histologic type grows very rapidly, infiltrating and occluding the dermal lymphatics, acute swelling and redness of the breast may develop, leading to a clinical picture referred to as inflammatory carcinoma. Inflammatory carcinoma, therefore, is not a true morphologic subtype but a clinical definition. Any form of breast cancer manifesting as “inflammatory carcinoma” is likely to be extremely aggressive. Medullary carcinoma has a slightly better prognosis than ductal or lobular types. It tends to occur in younger women, especially those with mutations of BRCA1 Grossly, this tumor is soft and well circumscribed. Histologically, this carcinoma is surrounded by a marked lymphoplasmacytic reaction, which may account for its propensity to grow more slowly than other forms of cancer. Medullary carcinoma, however, has a worse prognosis than tubular carcinoma. -
Question 51 of 100
51. Question
1 pointsA 7 year old child has a nearly confluent, line, erythematous, macular rash that is most pronounced on his trunk. He has had a mild fever for 36 hours but does not appear very sick, and is happily playing when the pediatrician enters the examining room. On examination findings are a reddened throat with tonsillar exudates, enlarged cervical nodes including the occipital node, and questionable splenomegaly. He has not been coughing. The most likely diagnosis is
Correct
Incorrect
Explanation:
This presentation of a fine, non blotchy rash which begins in the hairline and then spreads to the torso, in a child who is not very ill is characteristic of rubella, or German measles. The rash is associated with cervical lymphadenopathy. Koplik spots on the buccal mucosa are absent. IgM specific for rubella can often be detected in serum within 2 days of developing the rash. The principal significance of this disease is that it can cause a devastating congenital infection characterized by ocular problems (cataracts, retinopathy, microphthalmos, and glaucoma), cardiovascular problems (patent ductus arteriosus, ventricular septal defect, and pulmonary stenosis), deafness, thrombocytopenic purpura, hepatosplenomegaly, CNS problems, and bony lesions. Bullous pemphigoid produces large, tense blisters. Dermatitis herpetiformis causes recurrent crops of small vesicles or papules. Herpes simplex is characterized by crops of vesicles on oral or genital sites. Measles causes a blotchy, maculopapular erythematous rash that begins on the face and spreads downward. Patients with measles are usually much sicker than those with German measles, and Koplik spots maybe seen on the buccal mucosa.
-
Question 52 of 100
52. Question
1 pointsAnalysis of a man with excessive thirst and urination reveals serum osmolarity of 310 Osm/L and urine osmolarity of 90 mOsm/L. Plasma glucose is normal. Water deprivation fails to increase urine osmolarity. Vasopressin injection also fails to increase urine osmolarity. What is the most likely diagnosis?
Correct
Incorrect
Explanation:
Nephrogenic diabetes insipidus results in excessive excretion of free water in the urine because of an inability of the kidney to respond to vasopressin (antidiuretic hormone). It can occur in association with certain renal diseases that prevent the normal formation of the medullary concentration gradient. In this case, the kidney is unable to produce concentrated urine. Congenital nephrogenic diabetes insipidus could be due to a defect in the renal V2 receptor, Gs protein, or other steps in the normal formation of cyclic AMP. Plasma levels of vasopressin are usually increased because of the hyperosmolarity of the serum. Water deprivation will fail to increase urine osmolarity in both neurogenic and nephrogenic diabetes insipidus. However, subsequent injection of vasopressin will concentrate the urine in the case of neurogenic diabetes insipidus, but not in nephrogenic diabetes insipidus. Certain drugs (e.g. lithium) can also produce similar symptoms. Diabetes mellitus causes polyuria because of an osmotic diuresis due to glucosuria. This sort of diuresis does not involve the loss of much free water and the urine osmolarity lends toward that of the plasma (not 90 mOsm/L, as in the patient above). Furthermore, plasma glucose is normal in this patient, making diabetes mellitus unlikely. Some patients with primary hyperparathyroidism complain of increased urination. This is due to an osmotic diuresis, produced in this case by hypercalciuria. When serum levels of calcium exceed 12 mg/dL, the kidney´s ability to reabsorb filtered calcium is overwhelmed and hypercalciuria ensues. Primary polydipsia is a psychological disorder characterized by excessive water drinking. It too produces polyuria, but the excretion of free water is appropriate. These patients typically present with decreased serum osmolarity (due to the dilutional effect of the ingested water). Furthermore, water deprivation should produce concentrated urine. It is helpful to remember that the water deprivation test can distinguish between primary polydipsia and diabetes insipidus, but it cannot distinguish between neurogenic and nephrogenic diabetes insipidus.
-
Question 53 of 100
53. Question
1 pointsA 36-year-old male presents with oral and genital mucocutaneous ulcerations associated with Polyarthritis affecting the lower limbs. He is currently on warfarin for a recent episode of pulmonary embolism. Which of the genetic association is most commonly associated with his condition?
Correct
Incorrect
Explanation:
This is Behcet´s disease. It is associated with increased risk of thrombosis. It is linked to HLA B5. -
Question 54 of 100
54. Question
1 pointsThe most effective means of preventing viral hepatitis in a physician is
Correct
Incorrect
Explanation:
The greatest advance in prevention of hepatitis B has been the development of an effective vaccine which imparts immunity to close to 100% of those vaccinated. Following universal precautions and treatment with immune serum globulin enriched with anti-hepatitis B globulins can decrease the risk of hepatitis B transmission. Bleach is recommended as an irrigant following needle sticks. Unenriched immune serum globulin is only effective against hepatitis A.
-
Question 55 of 100
55. Question
1 pointsA 19-year-old has anemia and red cells have normal MCV but there is decreased total red cell counts. Serum iron is increased and TIBC is decreased. ESR, reticulocyte count and Hemoglobin structure are normal. What is the diagnosis?
Correct
Incorrect
Explanation:
Sideroblastic anemia often presents with normal mean corpuscular volumes but decreased total red blood cell counts. It can be differentiated on the basis of total serum iron levels, which are increased only in sideroblastic anemia and sometimes in anemia secondary to inheritance of thalassemia trait. However, sideroblastic anemia differs from thalassemia trait in that hemoglobin structure is normal, while in thalassemia the structure of hemoglobin represents an increase in beta chains and a decrease in alpha chains. Pernicious anemia produces megaloblasts.
-
Question 56 of 100
56. Question
1 pointsGibbous deformity is associated with which of the following disorder?
Correct
Incorrect
Explanation:
Tuberculosis can cause a gibbous (hunchback) deformity of the spine, which is much less common than it used to be. Ankylosing spondylitis, osteoarthritis, pneumococcus, and ulcerative colitis are not associated with this deformity. -
Question 57 of 100
57. Question
1 pointsObese adults with type II diabetes mellitus often have which of the following?
Correct
Incorrect
Explanation:
So called “adult onset” diabetes in overweight adults is often associated with increased insulin resistance. Such patients are not especially prone to diabetic ketoacidosis or diarrhea. Estrogen receptors have not been implicated in diabetes.
-
Question 58 of 100
58. Question
1 pointsAll of the following factors are associated with an increased risk of osteoporosis
Correct
Incorrect
Explanation:
Increased physical activity has been shown to decrease osteoporosis in older people. Calcium deficiency, cigarette smoking, corticosteroid usage, and a family history of osteoporosis are all risk factors for osteoporosis.
-
Question 59 of 100
59. Question
1 pointsA 44 year old female with multiple myeloma develops primary amyloidosis. Organ that is LEAST likely to be involved in the development of this disorder is
Correct
Incorrect
Explanation:
Primary amyloidosis, or amyloidosis that occurs as a result of multiple myeloma, is associated with macroglossia (tongue), peripheral neuropathy (nerves), arrhythmias (heart), and skin manifestations. Spleen involvement is seen in secondary amyloidosis.
-
Question 60 of 100
60. Question
1 pointsA person presents with headache, fever, and menigismus associated with a CSF pleoscytosis. Gram stain and cultures are negative and viral meningitis is diagnosed. True statement regarding viral meningitis is
Correct
Incorrect
Explanation:
Viral meningitis is usually benign and self-limiting, unlike bacterial meningitis which can be life threatening. Contacts of patients with bacterial meningitis, not viral meningitis, need antibiotic therapy. Antibiotics do not play a role in the management of viral meningitis.
-
Question 61 of 100
61. Question
1 pointsA male aged 58 years is diagnosed with Wegener´s granulomatosis. LEAST likely finding in this patient is which one of the following?
Correct
Incorrect
Explanation:
Wegener´s granulomatosis is an immune disorder associated with abnormal production of c-ANCA autoantibodies. The vasculitis typically affects small and medium sized vessels. However, there is often an accompanying necrotizing glomerulonephritis and necrotizing granuloma formation in the upper respiratory tract. The therapy for Wegener´s granulomatosis is immunosuppression. “Palpable purpura” is associated with hypersensitivity angiitis.
-
Question 62 of 100
62. Question
1 pointsA 14 year old boy has sore throat. Throat culture on sheep blood agar yields colonies of gram positive cocci that are surrounded by a zone of complete hemolysis. The organism it grew well on mannitol salt agar and caused the medium to turn yellow. The likely etiological agent of the patient´s condition is
Correct
Incorrect
Explanation:
The description of the agent is consistent with a staphylococcal organism (catalase positive gram positive cocci that grows on mannitol salt agar. The organism is most likely S.aureus, as it was able to ferment mannitol (as evidenced by the color change in the mannitol salt agar after incubation) and was beta hemolytic. Other characteristic of this organism is coagulase production. Staphylococcal organisms are also notorious for formation of abscesses; the patient had an abscess in the tonsillar region. The diphtheria bacilli (Corynebacterium diphtheria) are gram positive, pleomorphic rods that are arranged in palisades. They are non hemolytic and would not grow on mannitol salt agar. These organisms produce a whitish gray pseudomembrane on the pharynx or the larynx; constitutional symptoms from toxemia are the major features of diphtheria. Epiglottitis is the most common disease of the upper respiratory tract produced by Haemophilus influenzae, a gram negative, encapsulated rod. It is also a common cause of otitis media in children and may cause bronchitis, bronchiolitis, and pneumonia in adults. Streptococcus salivarius is a gram positive coccus that is usually alpha hemolytic. It is normal flora of the oral cavity and is sometimes implicated in subacute bacterial endocarditis, but is not associated with tonsillar abscesses. Streptococcus pyogenes is a beta hemolytic, gram-positive coccus that grows in chains, as opposed to the random, grape-like clusters of the staphylococci. These organisms are the most common cause of pharyngitis, which is usually manifested by severe sore throat, fever, a beefy red pharynx, and a tonsillar exudate.
-
Question 63 of 100
63. Question
1 pointsA 29-year-old lady presents with multiple cafe-au-lait spots. A diagnosis of neurofibromatosis type 1 is made. What is true of the NF1 gene?
Correct
Incorrect
Explanation:
Neurofibromatosis due to NF1 is found on chromosome 17 and is inherited as an autosomal dominant fashion. NF2 is associated with acoustic neuromas and is found on Chromosome 22. -
Question 64 of 100
64. Question
1 pointsThe level of cellular telomerase activity will affect:
Correct
Incorrect
Explanation:
The telomere is a DNA sequence at the end of each chromosome which becomes progressively shorter with each division the cell undergoes. When it is reduced to a critical length the cell is not capable of dividing, the enzyme telomerase is able to lengthen the telomere thus preventing this occurring.
-
Question 65 of 100
65. Question
1 pointsA 63 year old man undergoes screening colonoscopy, which demonstrates a 7 mm-diameter polyp in the rectum. Biopsy shows glands and sawtooth crypts composed of a proliferation of goblet and columnar epithelial cells. No cellular atypia is seen. What would be the best classification of this polyp?
Correct
Incorrect
Explanation:
This is a hyperplastic polyp; these polyps comprise 90% of all colonic polyps and have no malignant potential.
Peutz-Jeghers polyps (choice B) also have no malignant potential, but tend to be larger and hive a complex branching pattern.
Tubular adenomas, tubulovillous adenomas, and villous adenomas (choices C, D, and E) are all true neoplastic polyps containing dysplastic epithelium; the malignant potential of these polyps increases with size and the percentage of the polyp that has a villous configuration. -
Question 66 of 100
66. Question
1 pointsArsenic poisoning can best be detected in which one of the following?
Correct
Incorrect
Explanation:
Both nails and hair clippings can demonstrate arsenic for months after exposure. Urine, gastric contents, blood, and skin are not good sites for arsenic deposition.
-
Question 67 of 100
67. Question
1 pointsThe clinical presentation most consistent with vitamin D deficiency in the aged is
Correct
Incorrect
Explanation:
Vitamin D deficiency is being recognized more frequently among the elderly, especially in the nursing home or other settings where inactivity, nutritional deficiency, and lack of sunlight exposure combine to Increase the propensity for deficiency. Vitamin D deficiency has been associated with abnormalities in bone metabolism, primarily osetomalacia. Clinically this can present as muscle weakness, limb pain, and impaired physical function. Bleeding gums and fatigue may be presenting symptoms of scurvy (vitamin C deficiency). High-output cardiac failure with vasodilation, dermatitis, and neuropathy is the clinical presentation of thiamine deficiency. The combination of diarrhea, scaly dermatitis, and dementia is one of the clinical presentations of niacin deficiency
-
Question 68 of 100
68. Question
1 pointsOut of the following, which condition is caused by a bacterial infection?
Correct
Incorrect
Explanation:
Impetigo is caused by Group A beta-hemolytic streptococci or by Staphylococcus aureus. Tinea versicolor is a fungal disease. Lichen planus is an idiopathic condition unassociated with any infection. Psoriasis is an autoimmune form of skin rash. Seborrheic dermatitis is idiopathic and not associated with infection.
-
Question 69 of 100
69. Question
1 pointsA 54 year old man with occult blood in his stool has pedunculated villous adenomas in the proximal colon. Which of the following is NOT a risk factor for carcinoma of the colon?
Correct
Incorrect
Explanation:
Colon cancer is the third most common type of cancer in men. It can be cured if detected early by screening. Excess fats and colonic bacteria produce toxic bile acids, which predipose the colonic mucosa to tumor growth. A diet high in fiber has been shown to lower the risk of colon cancer. A family history of colon cancer is a risk factor for colon cancer. Increasing age, greater than 50 years, is associated with a risk for colon cancer. Rectal screening for occult blood should occur at age 40 and above. Proctosigmoidoscopy should occur starting at age 50 and every five years subsequent to that. Familial adenomatous polyposis predisposes to colon cancer and occurs in the third to fourth decade. It is recommended that prophylactic colectomy be undertaken in many cases. There is an increase in mucosal cell turnover which predisposes to tumor growth. Gardner´s syndrome is a familial disorder. It is associated with benign soft tissue tumors and osteomas. The associated colonic polyps may have malignant potential.
-
Question 70 of 100
70. Question
1 pointsJoint hypermobility is a feature of which one of the following?
Correct
Incorrect
Explanation:
A common manifestation of Ehlers Danlos syndrome is joint hypermobility. Joints tend to have reduced mobility in osteoarthritis and rheumatoid arthritis, and do not have increased mobility in chondroepiphyseal dysplasia or osteogenesis imperfecta.
-
Question 71 of 100
71. Question
1 pointsFALSE relationship between adenomatous colon polyps and carcinoma of the colon is
Correct
Incorrect
Explanation:
The highest malignancy rate is associated with a villous growth pattern. Adenomas are rare in countries with a low incidence of cancer. Adenomas and adenocarcinomas are located in the same anatomic location in the colon. In persons with adenomas, immature cells are found higher up the colonic crypt than normal. Although less than 5% of adenomas develop into carcinomas, the frequency of cancer in the polyp increases from 3% in lesions less than one centimeter in size to more than 40% in polyps larger than two centimeters.
-
Question 72 of 100
72. Question
1 pointsAll of the following are consequences of obstruction of the urinary tract, EXCEPT
Correct
Incorrect
Explanation:
Hypertension, due to increased release of renin by the involved kidney, is common in acute and subacute unilateral obstruction. Pain resulting from obstruction of the urinary tract results from distention of the collecting system or renal capsule. Azotemia occurs when overall excretory function is impaired. Both polyuria and nocturia, which do not respond to exogenous vasopressin, result from impaired renal concentrating ability. Partial bilateral obstruction may produce acquired distal renal tubular acidosis. This results from defects in tubule function due to renal tubulointerstitial damage.
-
Question 73 of 100
73. Question
1 pointsWhat autoantibodies are found at the highest frequency in Wegener´s granulomatosis?
Correct
Incorrect
Explanation:
Anti-neutrophil cytoplasmic antibodies are found in 95% of cases of Wegener´s granulomatosis.
-
Question 74 of 100
74. Question
1 pointsWhat autoantibodies are found at the highest frequency in Wegener´s granulomatosis?
Correct
Incorrect
Explanation:
Anti-neutrophil cytoplasmic antibodies are found in 95% of cases of Wegener´s granulomatosis.
-
Question 75 of 100
75. Question
1 pointsWhich of the following is NOT a complication of Paget´s disease?
Correct
Incorrect
Explanation:
Despite the massive increase in bone turnover, extracellular calcium homeostasis is usually normal. The plasma levels are thought to be regulated by reutilization of calcium ions as well as by feedback control of parathyroid secretion. Due to the marked proliferation of blood vessels in pagetic bone, blood flow to the affected areas may be significantly increased when the disease affects more than one third of the skeleton. This increased blood flow may be associated with high output cardiac failure. Pathologic fractures may occur during the destructive phase of the disease and is particularly common in persons with bowing. Urolithiasis probably results from increased urinary calcium excretion during the periods of bone resorption. Hearing loss may result from direct involvement of the ossicles of the inner ear or of bone in the region of the cochlea. Impingement by bone on the 8th cranial nerve as it passes through the auditory foramen may also result in hearing loss. -
Question 76 of 100
76. Question
1 pointsA 4 year old Somalian child´s evelids are irritated and inflamed. The tissue swelling is beginning to cause entropion. Scraping of the affected epithelium reveals iodine staining, intracytoplasmic inclusion bodies. This infectious agent transferred from human to human in which form?
Correct
Incorrect
Explanation:
Chlamydia trachomatis (serotypes A, B, Ba, and C) causes a conjunctival and corneal infection that is spread in developing countries by eye seeking flies. The lesions begin with formation of lymphoid follicles in the conjunctiva. With disease progression there is tissue necrosis, granulation tissue deposition, and scar formation, leading to lacrimal duct obstruction and distortion of the eyelids. The organism is an obligate intracellular parasite spread from human to human in a metabolically inactive extracellular form called an elementary body. Gram negative diplococci would be the transmission form if the child had acquired a Neisseria gonorrhoeae eye infection. These infections are transmitted to infants during passage through an infected birth canal. It would not be expected in this age group of children. Gram-positive cocci would be the transmission form if the child had a staphylococcal stye. These are localized abscesses that would not cause the formation of intracellular inclusion bodies. Naked capsid DNA virus would be the transmission form if the child had adenovirus conjunctivitis. In this case, the child would have conjunctival inflammation and injection, and intracellular inclusion bodies would be found in the affected tissues, not iodine staining intracytoplasmic inclusions. The reticulate form is the intracellular form of Chlamydia trachomatis. It is not the transmission, but the diagnostic, form.
-
Question 77 of 100
77. Question
1 pointsA 32 year old man intermittent abdominal pain. No pathology is seen on Esophagogastroduodenoscopy. Physician suspects that the patient may have a Meckel´s diverticulum, with a heterotopic rest of gastric mucosa. The Meckel´s diverticulum would most likely be found in which of the following areas
Correct
Incorrect
Explanation:
Meckel´s diverticulum is a common congenital anomaly that occurs when there is a persistence of the omphalomesenteric or vitelline duct. This duct initially connects the primitive gut to the yolk sac. Meckel´s diverticulum is usually found in the distal ileum within 12 inches of the ileocecal valve. It may contain heterotopic rests of gastric mucosa or pancreatic tissue. The bladder can develop acquired diverticula, typically secondary to over distension related to urinary tract obstruction. The duodenum does not usually develop diverticula. The esophagus can develop acquired diverticula typically as a consequence of scarring, with traction of its outer surface. Acquired diverticula of the gallbladder are common in cholelithiasis, and are sometimes called Rokitansky Aschoff sinuses.
-
Question 78 of 100
78. Question
1 pointsMechanism of drug resistance that is most likely to be transferred after an accident of the life cycle of a virulent phage?
Correct
Incorrect
Explanation:
Mutated penicillin binding proteins encoded on the chromosome are believed to be transferred from one Staphylococcus aureus organism to another via transduction. The process of generalized transduction is transfer following an accident of the life cycle of avirulent phage. Haemophilus influenzae ampicillin resistance is a plasmid encoded trait that is believed to be transferred from cell to cell during conjugation. It involves the production of a beta lactamase enzyme that inactivates the drug. Conjugation is a mechanism of genetic exchange between bacteria that involves cell-to-cell contact, but does not involve bacteriophages. Neisseria gonorrhoeae beta-lactamase production is a plasmid encoded trait that is transferred during conjugation using nonconjugative plasmids. This means that the beta lactamase enzyme genes are encoded on a plasmid that has lost its tra operon but retains oriT. The drug resistance plasmid can be mobilized across the conjugal bridge formed by another plasmid in the cell that has the tra operon. Staphylococcus aureus vancomycin resistance is a plasmid encoded gene that causes the production of a ligase enzyme that changes cell wall pentapeptides so that they will not bind the drug. This resistance is transferred during conjugation. Streptococcus pneumoniae penicillin binding protein mutations are chromosomal genes that are believed to be transferred during transformation. Transformation is the uptake and incorporation of free DNA by competent cells, followed by homologous recombination to stabilize the new genes.
-
Question 79 of 100
79. Question
1 pointsA 7 month old child is hospitalized for a yeast infection that does not respond to therapy. He has a history of multiple acute pyogenic infections. On examination the spleen and lymph nodes are not palpable. A differential WBC count shows 95 neutrophils, 1 lymphocyte, and 4 monocytes. Bone marrow biopsy contains no plasma cells or lymphocytes. CXR reveals the absence of a thymic shadow. Tonsils are absent. The most common cause of this condition is
Correct
Incorrect
Explanation:
Severe combined immunodeficiency (SCID) is associated with deficiencies in both Band T cells due to a defect in differentiation of an early stem cell. Over 50 % of the cases are caused by a gene defect on the X chromosome, resulting in a defective IL-2 receptor. Adenosine deaminase deficiency is an autosomal recessive disease characterized by a deficiency of adenosine deaminase, which results in accumulation of metabolites that are toxic to both B and T stem cells in the bone marrow. Children with severe infections usually die within the first 2 years of life unless they receive bone-marrow transplants. A defective CD40L molecule causes hyper-IgM syndrome. This makes it impossible for B lymphocytes to receive the signal to switch isotypes, and they are therefore “stuck” making IgM alone. It is not classified as a SCID. Inability to produce MHC-II causes bare lymphocyte syndrome, which causes patients to be unable to produce CD4 cells. It results in a type of SCID since TH cells are necessary for the function of all immune responses. Nonsense mutation of rag genes results in a SCID because without the ability to perform VDJ recombinations, lymphocytes cannot express surface receptors for antigen; and in that case, lymphocyte precursors are induced to undergo apoptosis in the bone marrow. Recombination activating genes (rag genes) are responsible for the VDJ rearrangements that allow lymphocytes to create the idiotype of their antigen receptors.
-
Question 80 of 100
80. Question
1 pointsMany encapsulated organisms are capable of entering the body across the respiratory mucosa. If these organisms enter the bloodstream, delivery to the meninges is a risk. Streptococcus pneumoniae, Hemophilus influenzae Neisseria meningitidis, and Cryptococcus neoformans are all capable of making this transition. The most important immunologic protective mechanism against such blood borne encapsulated organisms is
Correct
Incorrect
Explanation:
IgG is the predominant antibody in the serum. Encapsulated organisms are resistant to phagocytosis, but opsonization (enhancement of phagocytosis with IgG and/or C3b) is the most important mechanism for removing these pathogens from the bloodstream in the spleen. The spleen plays the critical role of slowing down the blood flow to allow the potential for cell-cell interactions, and opsonizations makes the engulfment of slippery encapsulated organisms occur up to 4000 times faster than in the absence of antibody and/or complement. C3 nephritic factor is an IgG autoantibody that binds to C3 convertase, making it resistant to inactivation. This leads to persistently low serum complement levels and is associated with Type II membranoproliferative glomerulonephritis. C5-9 mediated lysis is a secondary mechanism of destruction of encapsulated bacteria, because the capsule tends to protect against complement-mediated lysis. C5-9 is also known as the membrane-attack complex of complement.
Myeloperoxidase in phagocytic cells is an element of the oxygen-dependent intracellular pathway that effectively kills ingested organisms. The most efficient bactericidal system in phagocytes is dependent on NADPH oxidase, and the myeloperoxidase system is the next most powerful. Secretory IgA is the immunoglobulin associated with mucous membranes. Although the pathogens mentioned all enter the body across the mucous membranes, the question asks about the most important protective mechanism once they enter the blood. -
Question 81 of 100
81. Question
1 pointsA 33 year old woman gives birth to a 7 lb, 3 oz baby at term after a normal pregnancy. A large, red, raised discoloration of the infant´s face is observed in the delivery room that persists into adulthood. This is likely a component of which syndrome?
Correct
Incorrect
Explanation:
The lesion is a port wine stain, a vascular lesion that, unlike the more common strawberry nevus, does not usually regress with age. Port wine stains can be a component of Sturge Weber disease, which can also include similar vascular lesions of the meninges (leptomeningeal angiomatosis) and sometimes cutaneous angiomatosis at other sites. CT of the brain demonstrates a characteristic pattern of hemispheric atrophy thought to be due venous infarction, with tram-track calcification along the cortical ribbon.
Arnold-Chiari malformations are a cluster of related brain and spinal cord malformations in which there is a downward displacement of the cerebellar vermis and tonsils into the foramen magnum. Dandy-Walker malformation includes a distended fourth ventricle with a hypoplastic (or absent) cerebellum.
Neurofibromatosis includes benign and malignant peripheral nerve lesions and café au-lait spots. Tuberous sclerosis includes cortical tubers, adenoma sebaceum of the skin, pancreatic cysts, renal angiomyolipomas, and cardiac rhabdomyomas. -
Question 82 of 100
82. Question
1 pointsAnalysis of a man with excessive thirst and urination reveals serum osmolarity of 310 Osm/L and urine osmolarity of 90 mOsm/L. Plasma glucose is normal. Water deprivation fails to increase urine osmolarity. Vasopressin injection also fails to increase urine osmolarity. What is the most likely diagnosis?
Correct
Incorrect
Explanation:
Nephrogenic diabetes insipidus results in excessive excretion of free water in the urine because of an inability of the kidney to respond to vasopressin (antidiuretic hormone). It can occur in association with certain renal diseases that prevent the normal formation of the medullary concentration gradient. In this case, the kidney is unable to produce concentrated urine. Congenital nephrogenic diabetes insipidus could be due to a defect in the renal V2 receptor, Gs protein, or other steps in the normal formation of cyclic AMP. Plasma levels of vasopressin are usually increased because of the hyperosmolarity of the serum. Water deprivation will fail to increase urine osmolarity in both neurogenic and nephrogenic diabetes insipidus. However, subsequent injection of vasopressin will concentrate the urine in the case of neurogenic diabetes insipidus, but not in nephrogenic diabetes insipidus. Certain drugs (e.g. lithium) can also produce similar symptoms. Diabetes mellitus causes polyuria because of an osmotic diuresis due to glucosuria. This sort of diuresis does not involve the loss of much free water and the urine osmolarity lends toward that of the plasma (not 90 mOsm/L, as in the patient above). Furthermore, plasma glucose is normal in this patient, making diabetes mellitus unlikely. Some patients with primary hyperparathyroidism complain of increased urination. This is due to an osmotic diuresis, produced in this case by hypercalciuria. When serum levels of calcium exceed 12 mg/dL, the kidney´s ability to reabsorb filtered calcium is overwhelmed and hypercalciuria ensues. Primary polydipsia is a psychological disorder characterized by excessive water drinking. It too produces polyuria, but the excretion of free water is appropriate. These patients typically present with decreased serum osmolarity (due to the dilutional effect of the ingested water). Furthermore, water deprivation should produce concentrated urine. It is helpful to remember that the water deprivation test can distinguish between primary polydipsia and diabetes insipidus, but it cannot distinguish between neurogenic and nephrogenic diabetes insipidus.
-
Question 83 of 100
83. Question
1 pointsAn African male aged 40 years presents with severe abdominal pain. He has sickle cell anemia and has had many episodes of this type of pain in his abdomen as well as his extremities. He is hydrated and given analgesics. Which organ is likely functionally and structurally atrophic in this patient?
Correct
Incorrect
Explanation:
In patients with chronic sickle cell disease, serial infarctions cause the spleen to become an atrophic vestige after a few years. The spleen receives its blood supply from a single source, and this lack of collateral circulation makes it prone to infarction from sickle cell disease. Patients become functionally asplenic, and are rendered susceptible to infection by encapsulated organisms. Pneumococcal vaccination is indicated. Colon is not involved in sickle cell disease. The liver is incorrect. Patients with sickle cell disease tend to have hepatomegaly, and gallstones are common. Portal vein is uninvolved in sickle cell disease. The stomach is not involved in sickle-cell disease
-
Question 84 of 100
84. Question
1 pointsAn African boy develops massive unilateral enlargement of his lower face in the vicinity of the mandible. Biopsy reveals sheets of medium sized blast cells with admixed larger macrophages. This type of tumor has been associated with which one of the following?
Correct
Incorrect
Explanation:
The patient has Burkitt lymphoma. This high grade B cell lymphoma occurs endemically in Africa (it is the most common neoplasm in children in an equatorial belt that includes Africa and New Guinea) and sporadically in the U.S. and Europe. The sporadic form is often in an abdominal site and occurs in young adults. The African form of Burkitt lymphoma has been strongly associated with antibodies directed against Epstein Barr virus; the association is weaker in sporadic cases. A characteristic translocation,
t(8;14) (q24.13;q32.33) has been described. Malaria infection is a cofactor.
Hepatitis B is associated with hepatocellular carcinoma. t(9;22) is the Philadelphia chromosome, which is seen in some cases of chronic myelogenous leukemia (CML) and acute myelogenous leukemia (AML). Herpesvirus type 8 is associated with Kaposi sarcoma. CD5 is a marker seen in small lymphocytic and mantle cell lymphomas. HIV is linked to AIDS. Some patients also develop primary lymphomas (not usually Burkitt). CD4 is a marker for helper T cells and some T cell lymphomas. Human papillomavirus is linked with common warts, genital condylomata, and genital cancers. t(2;5) is linked to anaplastic large cell lymphoma. -
Question 85 of 100
85. Question
1 pointsA man is shot in head. The bullet enters his temple and severs his right optic nerve completely. He is quickly brought to ER and the physician tests his pupillary response by shining a light in the right eye. The physician will most likely find which one of the following?
Correct
Incorrect
Explanation:
This person is blind in the right eye. The afferent limb of the pupillary light reflex is carried by the optic nerve (CNII), and the efferent limb is via the oculomotor nerve (CN III), which carries parasympathetic fibers from the Edinger Westphal nucleus. Thus, shining a light in the affected eye will not elicit any pupillary response. On the other hand shining the light in the left eye will result in simultaneous constriction of both pupils (assuming an intact right CN III), since the left optic nerve is intact.
-
Question 86 of 100
86. Question
1 pointsA child is diagnosed with idiopathic thrombocytopenic purpura. The mother is worried if her child will develop serious bleeding. When reassuring this mother, it might be helpful to inform her that the threshold for generalized bleeding is about how many platelets/mm3?
Correct
Incorrect
Explanation:
Normal platelet counts are in the range of 150,000to 450,000/mm3 it is useful to know that peripheral blood platelet counts less than about 15,000-20,000/mm3 are required before generalized bleeding usually occurs. Few physicians choose to actively treat patients who are not currently bleeding and whose platelet counts are greater than 100,000/mm3. Clearly, patients with counts less than 15,000/mm3 probably require active therapy, possibly with platelet transfusions. However, there is more disagreement about how to handle patients with counts in the 20,000-50,000/mm3 range, and you may encounter some physicians who will attempt to transfuse these patients. (However, this practice is no longer recommended by the American Blood Bank Association unless the patient is actively bleeding.) Idiopathic thrombocytopenic purpura is due to an autoimmune attack on the platelets, leading to their clearing by the spleen. Splenectomy may be helpful in refractory cases. A count of 5,000 platelets/mm3 is dangerously low, and platelet transfusion should have been considered earlier. Some physicians will give platelet transfusions for actively bleeding patients with counts of 50,000/mm3. Platelet counts of 100,000/mm3 do not usually require platelet transfusion, even when bleeding. Platelet counts of 150,000/mm3 do not usually require platelet transfusion, even when bleeding.
-
Question 87 of 100
87. Question
1 pointsA 66 year old type 2 diabetic undergoes right above the knee amputation because of extensive leg gangrene. H&E stained cross section of the popliteal artery shows severe atherosclerotic changes with multifocal calcification of the media. According to current theories, what is the initial alteration leading to this condition?
Correct
Incorrect
Explanation:
The intima is markedly thickened and disrupted, the media contains massive foci of calcification and the lumen is narrowed. What is the initial alteration leading to such dramatic pathology? Current views on the pathogenesis of atherosclerosis indicate that the most likely initial step is injury to the endothelial cells, which can be due to a number of factors. High blood pressure nicotine immune mechanisms and hyperlipidemia are among the most important factors implicated in endothelial injury. This leads to accumulation of plasma derived lipids and proteins within the intima, with subsequent formation of an atheroma (atherosclerotic plaque). Collagen and elastin production, as well as proteoglycan synthesis are fundamental in the development of atheromatous lesions. This intercellular matrix often undergoes secondary calcification. Modified smooth muscle cells and fibroblasts produce these components.
Monocyte immigration is an essential step in atherosclerosis. Monocytes immigrating into the intima from the blood become macrophages and take up cholesterol bound to LDLs. Thus cholesterol laden monocytes with characteristic vacuolated cytoplasm accumulate in the center of the atheroma as “foam cells.” They secrete several cytokines (IL-1 and TNF-alpha) that stimulate fibroblasts and smooth muscle cells to proliferate and produce collagen elastin, and proteoglycans. Oxidation of low-density lipoproteins (LDLs) takes place as a result of free radicals released by monocytes and endothelial cells. Oxidized LDLs are taken up by macrophages through the scavenger receptor (separate from LDL receptor) and act as potent stimuli for production of IL-I and TNF-alpha. Platelet adhesion is a consequence of endothelial injury. Microthrombi develop on the endothelial surface, acting as a source of platelet derived growth factor (PDGF). PDGF attracts monocytes and stimulates smooth muscle cell proliferation. Smooth muscle cell proliferation is a response to various types of injury. In atherosclerosis, smooth muscle cells migrate from the media to the intima and transform into secretory cells that produce collagen elastin, and proteoglycans. PDGF, IL-I and TNF-alpha are important mediators of this process. -
Question 88 of 100
88. Question
1 pointsA chronic alcoholic male presents with confusion, ataxia, and nystagmus. Administration of thiamine leads to rapid symptomatic improvement. Neuropathologic change that accounts for this presentation is
Correct
Incorrect
Explanation:
The clinical picture of confusion, ataxia, and variable disturbances of eye movements (e.g., nystagmus and ophthahnoplegia) is due to Wernicke encephalopathy, which results from thiamine deficiency. Hemorrhage and necrosis in the above mentioned areas are the underlying pathologic changes. Alcoholics are particularly prone to this condition. Axonal degeneration of posterior and lateral columns of spinal cord, or subacute combined degeneration, results from two different etiologies: vitamin B12 deficiency and HIV myelopathy. Ataxia, numbness, and spastic paresis of the lower extremities are the clinical manifestations. Demyelination affecting the central region of the basis pontis is known as central pontine myelinolysis. This complication is most commonly encountered in severely malnourished and dehydrated alcoholics. Rapid correction of hyponatremia triggers this condition. Demyelination involving the corpus callosum and anterior commissure, or Marchiafava Bignami disease, is a rare disorder associated with chronic alcoholism. It is probably related to some dietary deficiency. Loss of neurons in the neocortex, cerebellar Purkinje cells, and hippocampal pyramidal neurons results from hypoxia or hypoglycemia related necrosis. These neurons are the most vulnerable to hypoxic or hypoglycemic injury.
-
Question 89 of 100
89. Question
1 pointsA tall man with gynecomastia and testicular atrophy has a testicular biopsy that shows sparse, completely hyalinized seminiferous tubules with a complete absence of germ cells and only rare Sertoli cells. Leydig cells are present in large clumps between the hyalinized tubules. Which genetic disorder should be suspected?
Correct
Incorrect
Explanation:
The testicular changes described are those observed in Klinefelter syndrome, most often due to 47, XXY genetics. Testicular feminization syndrome is due to a genetically determined unresponsiveness to testosterone that produces a phenotypic female in an individual with 46, XY chromosomes. Trisomy 18 is Edwards syndrome, characterized by facial features that are small and delicate. Trisomy 21 is Down syndrome, the most common trisomy. Characteristics include oblique palpebral fissures, epicanthal folds, endocardial cushion defects, simian creases, and a high-arched palate, among other anomalies. Turner syndrome, 45, XO, produces a sterile but phenotypic female of short stature with webbing of the neck.
-
Question 90 of 100
90. Question
1 pointsA 55 year old type 2 diabetic has progressive pins and needles sensation and numbness in both feet. On exam he had a mild bilateral weakness of foot dorsiflexion, both ankle reflexes were absent and plantar responses were flexor. There was absent sensation to light tough to mid shin level with loss of joint position sensation in the toes and absent vibration sensation below the hip. He has no evidence of a retinopathy and nephropathy. The most likely diagnosis is
Correct
Incorrect
Explanation:
Diabetic peripheral neuropathy usually goes in parallel with retinopathy and nephropathy. It is also slowly progressive and affects mainly spinothalamic pathway. Alcohol induced peripheral neuropathy is also slowly progressive and affects mainly spinothalamic pathway. Vitamin B12 deficiency usually causes a more rapidly progressive neuropathy with dorsal column involvement (joint position and vibration involvement with sensory ataxia and pseudoathetosis of upper limbs)
-
Question 91 of 100
91. Question
1 pointsTrue statement regarding IGF-1 levels is which one of the following?
Correct
Incorrect
Explanation:
IGF-l concentrations are often increased in pregnancy. Reduced IGF-l is typically found in adult GHD. Cirrhosis of the liver is due to reduced synthesis, diabetes mellitus and starvation.
-
Question 92 of 100
92. Question
1 pointsA woman aged 32 years on holiday presents with a swollen left calf. She states that she has ´Factor V Leiden mutation.´ Which statement is the most appropriate description of the pathophysiology of this mutation?
Correct
Incorrect
Explanation:
In the normal person, factor V functions as a cofactor to allow factor X to generate the active form of an enzyme called thrombin. Thrombin in turn cleaves fibrinogen to fibrin, which polymerizes to form the dense meshwork that makes up the majority of a clot. Activated protein C (aPC) is a natural anticoagulant that acts to limit the extent of clotting by cleaving and degrading factor V. Factor V Leiden is an autosomal dominant condition in which the coagulation factor cannot be destroyed by aPC. Mutation of the gene encoding factor Va single nucleotide substitution of adenine for guaninechanges the protein´s 506th amino acid from arginine to glutamine. Since this amino acid is normally the cleavage site for aPC, the mutation prevents efficient inactivation of factor V. When factor V remains active, it facilitates overproduction of thrombin leading to excess fibrin generation and excess clotting.
-
Question 93 of 100
93. Question
1 pointsAn 18-year-old boy has learning difficulties and is seen in the genetics clinic as his maternal uncles also had learning difficulties. Examinations reveal that the patient has large ears and large testes. What is the most likely genetic diagnosis?
Correct
Incorrect
Explanation:
Fragile X Syndrome is also known as Martin-Bell syndrome. In addition to moderate to severe mental retardation, other characteristics of individuals with Fragile X syndrome may include:
- Large ears
- Macroorchidism
- Prognathism
- Speech delays
- Prominent forehead
- Double-jointedness
- Autistic symptoms and
- Occasional self-mutilation
- The face is typically long and narrow, with a high arched palate and large ears
- Otitis media, strabismus, and dental problems may be present
Other common characteristics include: - Hyperextensible joints
- Hypotonia and
- Heart problems including mitral valve prolapse
In males, abnormally large testes are a distinctive feature. - In young children:
- Delayed motor development
- Hyperactivity
- Behavioral problems
- Toe walking, and
- Occasional seizures can occur
-
Question 94 of 100
94. Question
1 pointsWhich of the following may be responsible for a hypokalaemic hypertension?
Correct
Incorrect
Explanation:
Liddle´s syndrome is typically associated with hypokalaemic hypertension and low renin and aldosterone concentrations – the so- called pseudo-hyperaldosteronism. Bartter´s syndrome is associated with hypokalaemia, though hypertension is not a feature.
In type IV RTA, there is a hyporeninaemichypoaldosteronism, which may also be produced with diabetic nephropathy, hence hyperkalaemia is more typical. -
Question 95 of 100
95. Question
1 pointsWhich of the following does NOT characterize pleural effusion formation?
Correct
Incorrect
Explanation:
Fluid is removed via the lymphatics located in the parietal pleura. A pleural effusion develops when fluid formation exceeds absorption. Normally fluid enters the pleural space via the capillaries in the parietal pleura. Fluid also may enter the pleural space from the interstitial spaces of the lung via the visceral pleura. Small holes in the diaphragm also permit fluid to move from the peritoneal cavity into the pleural space. The lymphatics are capable of absorbing 20 times more fluid than is normally formed.
-
Question 96 of 100
96. Question
1 pointsA 25 year old male has itching on his arms and face. Examination shows well circumscribed wheals with raised, erythematous borders and blanched centers. The type of hypersensitivity that is this patient most likely having is
Correct
Incorrect
Explanation:
Urticaria (hives) is a good example of a local anaphylaxis reaction which is classified as a type I hypersensitivity reaction. Type I hypersensitivity reactions involve preformed IgE antibody bound to mast cells or basophils, which release vasoactive and spasmogenic substances when they react with antigens. The eosinophil-mediated cytotoxicity against parasites is an example of antibody-dependent, cell-mediated cytotoxicity. Type II hypersensitivity is caused by autoantibodies against cells or tissues. When these antibodies activate complement or promote phagocytosis, the tissue is damaged. Type III hypersensitivity is caused by immune complex deposition in the small diameter blood vessels and the subsequent activation of complement and destruction of the vasculature. The tuberculin (PPD) reaction, used to test for tuberculosis exposure, is an example of delayed-type, or type IV, hypersensitivity.
-
Question 97 of 100
97. Question
1 pointsInvestigations of a 37 year old female with excessive thirst and urination for the past 4 weeks and normal appetite show mildly elevated glucose and glucagons. Examination reveals tenderness in the left upper quadrant and an erythematous necrotizing skin eruption on her legs. Radiographs show tumor in the pancreas. The cell that is responsible for this lesion is which one of the following?
Correct
Incorrect
Explanation:
The symptoms described are typical for a patient with glucagon excess. Glucagon is secreted by alpha cells of the pancreatic islets of Langerhans. Increased levels are rare and are usually due to carcinoma (70%) or adenoma (30%) of the islets. Two thirds of patients with carcinoma have liver metastasis at the time of diagnosis. Increased glucagon causes a syndrome similar to diabetes mellitus, due to antagonism of insulin effects. Patients also have migratory skin rashes alopecia, hyperpigmentation of the skin and glossitis. Diagnosis is made by measuring glucagon. The acinar cell, the main exocrine cell of the pancreas, contains digestive enzymes. Increases in the release of these enzymes (especially amylase and lipase) occur with pancreatitis. Acute pancreatitis is considered an emergent medical condition. Chronic pancreatitis is associated with fibrosis and atrophy of the acinar structures. Beta cells are responsible for insulin release. An adenoma of beta cells (insulinoma) would cause hyperinsulinism. 10% are solitary and 10% are multiple. Insulinomas cause hypoglycemia, dizziness, confusion, and excessive sweating. Glucose needs to be given promptly to avoid coma and death. Diagnosis is made by finding increased insulin and hypoglycemia. Delta cells are islet cells that secrete somatostatin. Tumors producing increased somatostatin are usually malignant. Clinically, a diabetes mellitus-like syndrome occurs along with diarrhea. Diagnosis is made from elevated serum somatostatin levels, but can be difficult because of the hormone´s short half-life. The G cell is the islet cell that secretes gastrin. Gastrin excess is usually associated with gastrinomas, 70% of which are malignant. Zollinger Ellison syndrome ensues, causing low gastric and duodenal pH, mucosal ulceration and diarrhea. Diagnosis is made from high serum gastrin levels.
-
Question 98 of 100
98. Question
1 pointsA 72 year old man is diagnosed with chronic renal failure. He has been instructed to visit a dietician. A summary of the dietician´s recommendations would likely include instructions that patient´s diet contain which combination?
Correct
Incorrect
Explanation:
In general, a “renal failure diet” is designed to control the amount of protein and phosphorus in patient´s diet. Furthermore, patients may be instructed to limit the amount of sodium and total fluid intake. In summary, a renal failure diet, provided there are no other contraindications to this recommendation should be low in protein (Options A, C, D and E are incorrect). Although the ingestion of 3 to 4L of fluid on a daily basis is recommended for adults with normal renal function, the GFR and subsequent elimination of water are decreased in patients with renal dysfunction. A patient with renal dysfunction and/or renal failure who ingests this volume of fluid on a daily basis is likely to develop peripheral edema. Therefore, a dietician would most likely recommend that a patient with chronic renal failure should have a diet; that consists of high carbohydrate and moderate fat intake as well as low amounts of sodium and phosphorus.
-
Question 99 of 100
99. Question
1 pointsA 16 year old girl with primary amenorrhea is short, has widely spaced nipples, and shows Tanner stage I breast and genitalia, and little pubic hair. Weak pulses are present in the lower extremities and serum levels of free T3 and T4 are decreased. The failure to go through puberty is directly related to which condition?
Correct
Incorrect
Explanation:
This is the classic presentation of Turner syndrome, which is the most common sex chromosome abnormality in females. It is due to complete or partial monosomy of the X chromosome (45, XO). More than half of cases are due to complete loss of the X chromosome (no Barr body is present in karyotype) while the remaining cases are mostly due to mosaicism. Turner syndrome is the most important cause of primary amenorrhea. Ninety percent of cases include dysgenesis of the ovaries; consequently, these patients lack the steroids required for breast development and cyclic function of the hypothalamus pituitary, ovaries, and uterus. This syndrome is frequently associated with congenital cardiac anomalies (aortic coarctation or bicuspid aortic valve), hypothyroidism, and diabetes. The most common type of coarctation of the aorta associated with Turner syndrome is the preductal type. The narrowing is proximal to the ductus arteriosus, and patients present with weak pulses hypotension, and cyanosis of the lower extremities. The chest X-ray shows rib notching (erosion of the lower border of the ribs from enlargement of the intercostals providing collateral blood flow) and right ventricular hypertrophy with increased pulmonary markings. Thyrotropin releasing hormone in excess increases prolactin sufficiently to suppress ovarian/uterine cycling, but it also tends to cause galactorrhea. It is not associated with Turner, and the evidence is that it is decreased in this patient. Ovarian steroids, including progesterone which has a contraceptive effect, are decreased due to ovarian dysgenesis. Hypothalamic disorders, such as deficient secretion of GnRH, are not the principal feature responsible for failure of menarche in Turner. Primary pituitary insufficiency is not typical, and in any case the failure of the gonads to develop is the key element of the amenorrhea.
-
Question 100 of 100
100. Question
1 pointsA boy aged 19 years presents with pneumonia. He has had recurrent pneumonia and sinusitis due to Streptococcus pneumoniae and Haemophilus influenza since the age of 6 months. Careful assessment of his immune function would likely reveal abnormal function of which one of the following?
Correct
Incorrect
Explanation:
The symptoms in the question suggest a deficiency known as common variable hypogammaglobulinemia, characterized by very low serum levels of IgG. In this congenital disease, the number of B cells is normal, but their ability to synthesize IgG and the other immunoglobulins is severely com-promised, leading to recurrent streptococcal and Haemophilus infections. The onset of the recurrent infections usually begins between 6 and 12 months of age concurrent with the decreasing levels of maternal IgG in the newborn. Treatment often involves giving IV gamma globulin to reduce the number of infections.
Abnormal function of macrophages such as that seen in chronic granulomatous disease results in recurrent infections with catalase positive organisms. Streptococcus is a catalase-negative genus. Abnormal function of NK cells is unlikely to present as recurrent bacterial infections, because these cells are primarily involved in cellular, rather than humoral, immunity. Platelets are not directly involved in the immune response. Deficiencies in platelets lead to problems in clotting and produce bleeding disorders. T-cell deficiencies result in severe viral, fungal, and protozoal infections, rather than recurrent bacterial infections.